LSAT and Law School Admissions Forum

Get expert LSAT preparation and law school admissions advice from PowerScore Test Preparation.

User avatar
 Dave Killoran
PowerScore Staff
  • PowerScore Staff
  • Posts: 5852
  • Joined: Mar 25, 2011
|
#59772
Complete Question Explanation
(The complete setup for this game can be found here: lsat/viewtopic.php?t=26553)

The correct answer choice is (E)

Since this is an Except question, four of the answer choices Could Be True, and the one correct answer choice Cannot Be True. The question states that G is assigned to bench 4, and X to bench 3. From the global conditions, we know that W cannot be assigned to the same bench as G, so she cannot be at bench 4. From the super-block, we know that V cannot be at bench 4. Since X is at bench 3, he cannot be at bench 4. This leaves Y as the only possible non-major to sit at bench 4, so she must be seated there, and we now know that bench 4 seats only G and Y. Thus, answer choice (E) is correct because J cannot be assigned to the same bench as Y.

If you do not see the inference pattern above, remember that you can make hypotheticals based on the limited positions of the VFJ super-block. In this case, the VFJ super-block has only two possible assignments: benches 1 and 2 or benches 2 and 3:
pt26_s98_g1_q5.png

Again, after the VFJ super-block is placed, the application of the GW not-block allows all the variables to be assigned. By comparing each answer choice against the two hypotheticals, you could still determine that answer choice (E) is correct.

Get the most out of your LSAT Prep Plus subscription.

Analyze and track your performance with our Testing and Analytics Package.